2021 AMC 10B Problems/Problem 16

Revision as of 16:05, 5 March 2021 by MRENTHUSIASM (talk | contribs) (Solution 4: Solution completed.)

Problem

Call a positive integer an uphill integer if every digit is strictly greater than the previous digit. For example, $1357, 89,$ and $5$ are all uphill integers, but $32, 1240,$ and $466$ are not. How many uphill integers are divisible by $15$?

$\textbf{(A)} ~4 \qquad\textbf{(B)} ~5 \qquad\textbf{(C)} ~6 \qquad\textbf{(D)} ~7 \qquad\textbf{(E)} ~8$

Solution 1

The divisibility rule of $15$ is that the number must be congruent to $0$ mod $3$ and congruent to $0$ mod $5$. Being divisible by $5$ means that it must end with a $5$ or a $0$. We can rule out the case when the number ends with a $0$ immediately because the only integer that is uphill and ends with a $0$ is $0$ which is not positive. So now we know that the number ends with a $5$. Looking at the answer choices, the answer choices are all pretty small, so we can generate all of the numbers that are uphill and are divisible by $3$. These numbers are $15, 45, 135, 345, 1245, 12345$ which are $6$ numbers C.

Solution 2

First, note how the number must end in either $5$ or $0$ in order to satisfying being divisible by $15$. However, the number can't end in $0$ because it's not strictly greater than the previous digits. Thus, our number must end in $5$. We do casework on the number of digits. $\newline$

Case 1 = $1$ digit. No numbers work, so $0$ $\newline$

Case 2 = $2$ digits. We have the numbers $15, 45,$ and $75$, but $75$ isn't an uphill number, so $2$ numbers. $\newline$

Case 3 = $3$ digits. We have the numbers $135, 345$. So $2$ numbers. $\newline$

Case 4 = $4$ digits. We have the numbers $1235, 1245$ and $2345$, but only $1245$ satisfies this condition, so $1$ number. $\newline$

Case 5= $5$ digits. We have only $12345$, so $1$ number. $\newline$

Adding these up, we have $2+2+1+1 = 6$. $\boxed {C}$

~JustinLee2017

Solution 3

Like solution 2, we can proceed by using casework. A number is divisible by $15$ if is divisible by $3$ and $5.$ In this case, the units digit must be $5,$ otherwise no number can be formed.

Case 1: sum of digits = 6

There is only one number, $15.$

Case 2: sum of digits = 9

There are two numbers: $45$ and $135.$

Case 3: sum of digits = 12

There are two numbers: $345$ and $1245.$

Case 4: sum of digits = 15

There is only one number, $12345.$

We can see that we have exhausted all cases, because in order to have a larger sum of digits, then a number greater than $5$ needs to be used, breaking the conditions of the problem. The answer is $\textbf{(C)}.$

~coolmath34

Solution 4

For every positive integer:

$\quad\bullet$ It is divisible by $3$ if and only if its digit-sum is divisible by $3.$

$\quad\bullet$ It is divisible by $5$ if and only if its units digit is $0$ or $5.$

$\quad\bullet$ It is divisible by $15$ if and only if it is divisible by both $3$ and $5.$

Since the desired positive integers are going uphill, their units digits must be $5$s. We start with $12345,$ the largest of such positive integers, and perform casework on removing the number of its digits. Clearly, we cannot remove the digit $5,$ as that is the only way to satisfy the divisibility of $5.$ Now, we focus on the divisibility of $3.$

Case (1): Remove exactly $\boldsymbol{0}$ digits. ($\boldsymbol{5}$-digit numbers)

The number $12345$ has a digit-sum of $15.$ So, it is divisible by $3.$

There is $1$ uphill integer in this case: $12345.$

Case (2): Remove exactly $\boldsymbol{1}$ digit. ($\boldsymbol{4}$-digit numbers)

We can only remove the digit $3.$

There is $1$ uphill integer in this case: $1245.$

Case (3): Remove exactly $\boldsymbol{2}$ digits. ($\boldsymbol{3}$-digit numbers)

We can only remove the digits that sum to a multiple of $3.$ This sum can only be $3$ or $6$ here.

There are $2$ uphill integers in this case: $345,135.$

Case (4): Remove exactly $\boldsymbol{3}$ digits. ($\boldsymbol{2}$-digit numbers)

We can only remove the digits that sum to a multiple of $3.$ This sum can only be $6$ or $9$ here.

There are $2$ uphill integers in this case: $45,15.$

Case (5): Remove exactly $\boldsymbol{4}$ digits. ($\boldsymbol{1}$-digit numbers)

As discussed above, we must keep the digit $5.$ Since $5$ is not divisible by $3,$ this case is impossible.

Total

Together, our answer is $1+1+2+2=\boxed{\textbf{(C)} ~6}.$

~MRENTHUSIASM

Video Solution by OmegaLearn (Using Divisibility Rules and Casework)

https://youtu.be/n2FnKxFSW94

~ pi_is_3.14

Video Solution by TheBeautyofMath

https://youtu.be/FV9AnyERgJQ

~IceMatrix

Video Solution by Interstigation

https://youtu.be/9ZlJTVhtu_s

~Interstigation

See Also

2021 AMC 10B (ProblemsAnswer KeyResources)
Preceded by
Problem 15
Followed by
Problem 17
1 2 3 4 5 6 7 8 9 10 11 12 13 14 15 16 17 18 19 20 21 22 23 24 25
All AMC 10 Problems and Solutions

The problems on this page are copyrighted by the Mathematical Association of America's American Mathematics Competitions. AMC logo.png